The ring Zp we have (a+b)^p =a^p + b^p for every a, b element of Zp. For Z6, the result is not true.
To show that (a+b)^p = a^p + b^p in the ring Zp, we can use the binomial theorem for a commutative ring.
For any integer n>=0 and elements a,b in a commutative ring, we have:
(a+b)^n = sum(k=0 to n) (n choose k) a^(n-k) b^k
where (n choose k) is the binomial coefficient.
Now, for p prime, the binomial coefficients (p choose k) are divisible by p for 0<k<p. This can be shown using the formula:
(p choose k) = p!/((p-k)! k!)
where p! is the factorial of p. Since p is prime, the denominator of (p choose k) contains no factors of p, but the numerator does, precisely p factors. Thus, (p choose k) is divisible by p for 0<k<p.
Using this fact, we can simplify the binomial expansion for (a+b)^p as follows:
(a+b)^p = sum(k=0 to p) (p choose k) a^(p-k) b^k
= a^p + b^p + pa^(p-1)b + pab^(p-1) + sum(k=2 to p-1) (p choose k) a^(p-k) b^k
Since (p choose k) is divisible by p for 0<k<p, we see that each of the terms in the sum is divisible by p, so we can conclude that:
(a+b)^p = a^p + b^p (mod p)
which means that (a+b)^p and a^p + b^p differ by a multiple of p, and hence they are equal in the ring Zp.
For Z6, the result is not true. For example, take a=2 and b=4. Then:
(a+b)^p = 6^p = 0 (mod 6)
but
a^p + b^p = 2^p + 4^p
can take different values modulo 6 depending on the value of p. For instance, when p=2 we have:
2^2 + 4^2 = 20 = 2 (mod 6)
but when p=3 we have:
2^3 + 4^3 = 72 = 0 (mod 6)
so in general a^p + b^p need not be equal to (a+b)^p in Z6.
Learn more about binomial at https://brainly.com/question/14526794
#SPJ11
Find the area of the part of the plane 3x 2y z = 6 that lies in the first octant.
The area of the part of the plane 3x 2y z = 6 that lies in the first octant is mathematically given as
A=3 √(4) units ^2
What is the area of the part of the plane 3x 2y z = 6 that lies in the first octant.?Generally, the equation for is mathematically given as
The Figure is the x-y plane triangle formed by the shading. The formula for the surface area of a z=f(x, y) surface is as follows:
\(A=\iint_{R_{x y}} \sqrt{f_{x}^{2}+f_{y}^{2}+1} d x d y(1)\)
The partial derivatives of a function are f x and f y.
\(\begin{aligned}&Z=f(x)=6-3 x-2 y \\&=\frac{\partial f(x)}{\partial x}=-3 \\&=\frac{\partial f(y)}{\partial y}=-2\end{aligned}\)
When these numbers are plugged into equation (1) and the integrals are given bounds, we get:
\(&=\int_{0}^{2} \int_{0}^{3-\frac{3}{2} x} \sqrt{(-3)^{2}+(-2)^2+1dxdy} \\\\&=\int_{0}^{2} \int_{0}^{3-\frac{3}{2} x} \sqrt{14} d x d y \\\\&=\sqrt{14} \int_{0}^{2}[y]_{0}^{3-\frac{3}{2} x} d x d y \\\\&=\sqrt{14} \int_{0}^{2}\left[3-\frac{3}{2} x\right] d x \\\\\)
\(&=\sqrt{14}\left[3 x-\frac{3}{2} \cdot \frac{1}{2} \cdot x^{2}\right]_{0}^{2} \\\\&=\sqrt{14}\left[3-\frac{3}{2} \cdot \frac{1}{2} \cdot x^{2}\right]_{0}^{2} \\\\&=\sqrt{14}\left[3.2-\frac{3}{2} \cdot \frac{1}{2} \cdot 3^{2}\right] \\\\&=3 \sqrt{14} \text { units }{ }^{2}\)
In conclusion, the area is
A=3 √4 units ^2
Read more about the plane
https://brainly.com/question/1962726
#SPJ4
which of the following represents the diameter of the circle below?
Answer:
\(\overline{TH}\)
Step-by-step explanation:
A - This is the radius of the circle (half the diameter, we are not looking for this)B - This is the center of the circle (a point)C - This is a point on the circle (also a point)D - This is the diameter of the circleThe diameter of a circle is a line that goes from one side to the other in a circle. It is the longest straight line that can be drawn in a circle. It also goes through the center of a circle.
That is the answer
- Kan Academy Advance
If you were to hypothesize that communication students will have a higher average score on the oral communication measures, you would have a ______.
If I were to hypothesize that communication students will have a higher average score on the oral communication measures,
I would have a research hypothesis. A research hypothesis is a statement that is used to explain a relationship between two or more variables,
in this case, the relationship between being a communication student and having a higher score on oral communication measures.
The hypothesis can then be tested through research and analysis of data to determine if there is a significant correlation between the two variables. In order to fully test this hypothesis,
it would be necessary to gather data on both communication students and non-communication students and compare their scores on oral communication measures.
learn more about communication here:brainly.com/question/22558440
#SPJ11
Fill in the blanks using < or >. (a) -3 ...... -4 (b) 6 ....... -20 (c) -8 ...... -2 (d) 5 ...... -7
\(\sf \bf {\boxed {\mathbb {ANSWER:}}}\)
(a) -3 \(\boxed{ > }\) -4
(b) 6 \(\boxed{ >}\) -20
(c) -8 \(\boxed{ < }\) -2
(d) 5 \(\boxed{ > }\) -7
\(\large\mathfrak{{\pmb{\underline{\orange{Mystique35 }}{\orange{☂}}}}}\)
may someone solve this please and show work, thank u
Answer:
minutes took to finish the race = 54 minutes
Step-by-step explanation:
It's known that:
\(speed = \frac{distance}{time}\)
then:
\(time = \frac{distance}{speed}\)
IN THE 1ST 10 MILES:
time = \(\frac{10}{25} = \frac{2}{5} = 0.4 hours\)
IN THE 2ST 10 MILES:
time = \(\frac{10}{20} = 0.5 hours\)
Hence, Overall time took to finish the race = 0.4 + 0.5 = 0.9 hours
time in minutes = 0.9 × 60 = 54 minutes
Another Solution:
Overall speed = \(\frac{S_1 + S_2}{2}\), Where: S1: 1st speed, S2: 2nd speed
then:
speed = \(\frac{25 + 20}{2}= 22.5 mph\)
Hence, time took = \(\frac{20}{22.5} = 0.9 hours = 54 minutes\)
hope you find this easy to understand....
Have any questions? Write in the comments.
give me brainliest if you found this answer useful
A man has 1/2 of a kilogram of pasta. He shares it equally amongst 4 people. How many kilograms does each person get? Input your answer as a fraction of a kilogram.
Answer:
1/8 kilgram of pasta. is how much each person will get.
Step-by-step explanation:
1/2 divided by four ... put bot into a common denomenator and then finish the problem. since they have same denomenator it should be easy
Use the Fundamental Theorem of Calculus to find the "area under curve" of \( f(x)=2 x+10 \) between \( x=3 \) and \( x=6 \). Answer:
The area under the curve of f(x) is 57.
The Fundamental Theorem of Calculus states that if a function f(x) is continuous on the interval [a, b] and F(x) is an antiderivative of f(x) on that interval, then the definite integral of f(x) from a to b is equal to F(b) - F(a):
∫[a, b] f(x) dx = F(b) - F(a).
In this case, we are given the function f(x) = 2x + 10 and we want to find the area under the curve between x = 3 and x = 6.
To use the Fundamental Theorem of Calculus, we need to find an antiderivative of f(x). The antiderivative of 2x is \(x^2\), and the antiderivative of 10 is 10x. Therefore, an antiderivative of f(x) = 2x + 10 is F(x) = \(x^2\) + 10x.
Now, we can apply the Fundamental Theorem of Calculus:
∫[3, 6] (2x + 10) dx = F(6) - F(3).
Evaluating F(x) at x = 6 and x = 3, we have:
F(6) = \((6)^2\) + 10(6) = 36 + 60 = 96,
F(3) = \((3)^2\) + 10(3) = 9 + 30 = 39.
Substituting these values into the equation, we get:
∫[3, 6] (2x + 10) dx = F(6) - F(3) = 96 - 39 = 57.
Therefore, the area under the curve of f(x) = 2x + 10 between x = 3 and x = 6 is 57.
To learn more about area here:
https://brainly.com/question/15122151
#SPJ4
suppose a clothing store wants to determine the current percentage of customers who are over the age of forty. how many customers should the company survey in order to be 98% confident that the estimated (sample) proportion is within 2 percentage points of the true population proportion of customers who are over the age of forty?
The company survey 307 customers in order to get 98% confidence.
Sample proportion (p) = 0.5 (as no estimate is given)
α = 1 - 92% = 0.08. Therefore, α/2 = 0.08/2 = 0.04
Z(α/2) = Z(0.04) = 1.751
Estimate = 5% = 0.05
Now, E = Z(α/2) * sqrt (p(1-p)/n)
n = (1.751/0.05)2 * 0.5 * (1-0.5)
n = 307
Hence, the company should survey 307 customers in order to be 98% confident that the estimated (sample) proportion is within 2 percentage points of the true population proportion of customers who are over the age of forty.
To know more about estimate questions:
https://brainly.com/question/23324288
#SPJ4
Find volume of one penny if volume of 50 pennies is 18.0 mL.
The volume of one penny is 0.36 mL if the volume of 50 pennies is 18.0 mL.
To find the volume of one penny if the volume of 50 pennies is 18.0 mL, we use the concept of proportionality as follows:
We can find the volume of one penny by dividing the volume of 50 pennies by 50 since we know that 50 pennies occupy a volume of 18.0 mL.
Therefore, the volume of one penny can be calculated as:Volume of one penny = Volume of 50 pennies / 50= 18.0 mL / 50= 0.36 mL
Hence, the volume of one penny is 0.36 mL if the volume of 50 pennies is 18.0 mL.
Learn more about volume at
https://brainly.com/question/20747240
#SPJ11
4 3/4 plus 1/3 what does it equal
In the coordinate plane, line p has slope 1/3 and y-intercept (0, 2). Line r is the result of dilating line p by a factor of 3 centered at the origin. What are the slope and y-intercept of line r?.
The line r has a slope 1/3 and y-intercept (0,5)
What is the Dilation?
Math definition of dilation Resizing an object is accomplished through a change called dilation. The objects can be enlarged or shrunk via dilation. A shape identical to the source image is created by this transformation. The size of the form does, however, differ.
Given
The slope of the line and its intercept (0,2)
The slope of the line r will not change.
And intercept is the addition of their ordinate will be.
Intercept = 2+3
Intercept = 5
So, the line r has slope 1/3 and y-intercept (0,5).
To learn more about the Dilation from the given link
https://brainly.com/question/3457976
#SPJ4
Let T be a linear transformation from R3 to R3 such that T(1,0,0)=(4,−1,2),T(0,1,0)=(−2,3,1),T(0,0,1)=(2,−2,0). Find T(1,0,−3).
Value of a linear transformation T(1,0,-3) is (-2, 7, -5).
Given a linear transformation T from R³ to R³ such that T(1, 0, 0) = (4, -1, 2), T(0, 1, 0) = (-2, 3, 1) and T(0, 0, 1) = (2, -2, 0), we are required to find T(1, 0, -3).
Given a linear transformation T from R³ to R³ such that T(1, 0, 0) = (4, -1, 2), T(0, 1, 0) = (-2, 3, 1) and T(0, 0, 1) = (2, -2, 0), we know that every element in R³ can be expressed as a linear combination of the basis vectors (1,0,0), (0,1,0), and (0,0,1).
Therefore, we can write any vector in R³ in terms of these basis vectors, such that a vector v in R³ can be expressed as v = (v1,v2,v3) = v1(1,0,0) + v2(0,1,0) + v3(0,0,1).
From this, we know that any vector v can be expressed in terms of the linear transformation
T as T(v) = T(v1(1,0,0) + v2(0,1,0) + v3(0,0,1)) = v1T(1,0,0) + v2T(0,1,0) + v3T(0,0,1).
Therefore, to find T(1,0,-3),
we can express (1,0,-3) as a linear combination of the basis vectors as (1,0,-3) = 1(1,0,0) + 0(0,1,0) - 3(0,0,1).
Thus, T(1,0,-3) = T(1,0,0) + T(0,1,0) - 3T(0,0,1) = (4,-1,2) + (-2,3,1) - 3(2,-2,0) = (-2, 7, -5).
Therefore, T(1,0,-3) = (-2, 7, -5).
Learn more about linear transformation
brainly.com/question/13595405
#SPJ11
Find the median of data 1/3 3/2 1/6 1/4 2/3
Answer:
median= 1/3
Step-by-step explanation:
A quality control expert at a drink bottling factory took a random sample of 9 bottles from a
production run of over 1,000 bottles and measured the amount of liquid in each bottle in the
sample. The sample data were roughly symmetric with a mean of 505 mL and a standard
deviation of 6 mL.
Based on this sample, which of the following is a 95% confidence interval for the mean
amount of liquid per bottle (in mL) in this production run?
Answer: It's in the screenshot
Step-by-step explanation:
Using the t-distribution, as we have the standard deviation for the sample, it is found that the 95% confidence interval for the mean amount of liquid per bottle (in mL) in this production run is (500.39, 509.61).
What is a t-distribution confidence interval?The confidence interval is:
\(\overline{x} \pm t\frac{s}{\sqrt{n}}\)
In which:
\(\overline{x}\) is the sample mean.t is the critical value.n is the sample size.s is the standard deviation for the sample.The critical value, using a t-distribution calculator, for a two-tailed 95% confidence interval, with 9 - 1 = 8 df, is t = 2.306.
The other parameters are given as follows:
\(\overline{x} = 505, s = 6, n = 9\)
Hence:
\(\overline{x} - t\frac{s}{\sqrt{n}} = 505 - 2.306\frac{6}{\sqrt{9}} = 500.39\)
\(\overline{x} + t\frac{s}{\sqrt{n}} = 505 + 2.306\frac{6}{\sqrt{9}} = 509.61\)
The 95% confidence interval for the mean amount of liquid per bottle (in mL) in this production run is (500.39, 509.61).
More can be learned about the t-distribution at https://brainly.com/question/16162795
Write f(x)=sprt(6x+7) as simpler function by introducing other function notations such as g(x) and h(x).
Answer:
\(f(x) = h(g(x))\)
Step-by-step explanation:
Given
\(f(x) = \sqrt{6x + 7}\)
Required
Write as simpler function
Let
\(h(x) = \sqrt x\)
\(g(x) = 6x + 7\)
Apply composite function rule:
\(h(x) = \sqrt x\)
\(h(g(x)) = \sqrt{g(x)}\)
Substitute: \(g(x) = 6x + 7\)
\(h(g(x)) = \sqrt{6x + 7}\)
By comparison:
\(f(x) = h(g(x)) = \sqrt{6x + 7}\)
So:
\(f(x) = h(g(x))\)
Where:
\(h(x) = \sqrt x\)
\(g(x) = 6x + 7\)
are natural numbers closed under subtraction
The natural numbers are "closed" under addition and multiplication. ... The subtraction of two natural numbers does NOT necessarily create another natural number (3 - 10 = -7). The division of two natural numbers does NOT necessarily create another natural number (1 ÷ 2 = ½). not closed under subtraction and division.
NO LINKS OR FILLES!!!!!! The capacity of the container is 1 gallon. If the amount of juice decreased by half, how much would remain?
A) 1 quart
B) 1 pint
C) 2 pints
D) 4 cups
The amount of juice decreased by half, how much would remain is 2 pints. So, the correct answer is C.
The capacity of the container is 1 gallon, which is equivalent to 4 quarts, 8 pints, or 16 cups. If the amount of juice decreases by half, you would have 50% of the original amount remaining.
To calculate this, you can use the following conversions:
1 gallon = 4 quarts
1 gallon = 8 pints
1 gallon = 16 cups
Half of 1 gallon is:
0.5 * 4 quarts = 2 quarts
0.5 * 8 pints = 4 pints
0.5 * 16 cups = 8 cups
Since 2 quarts is equivalent to 4 pints and 8 cups, the correct answer is:
A) 1 quart (2 quarts remaining)
B) 1 pint (4 pints remaining)
C) 2 pints (4 pints remaining)
D) 4 cups (8 cups remaining)
The correct answer is C) 2 pints, as 4 pints of juice would remain in the container after decreasing the amount by half.
Learn more about the capacity at
https://brainly.com/question/30205394
#SPJ11
The table below shows the ratios of black to white keys on pianos of various sizes.
Black A 18 45 72
White 13 B 65 C
Determine which table has the correct values for A, B, and C.
Black 8 18 45 72
White 13 25 65 105
Black 9 18 45 72
White 13 26 65 104
Black 9 18 45 72
White 13 39 65 117
Black 1 18 45 72
White 13 26 65 104
Using the given ratios found the values of A, B and C, and completed the table as follows:
Black 9 18 45 72
White 13 26 65 104
What is meant by the ratio?
A ratio in mathematics demonstrates how many times one number is present in another. A ratio is what a fraction looks like when it is written as a:b. A ratio is the comparative or condensed version of two quantities of the same type.
Given the table:
Black A 18 45 72
White 13 B 65 C
The table represents the ratios of black keys to white keys on pianos of various sizes.
The ratio of keys of the piano always remains constant.
So we can write as follows:
A/13 = 18/B = 45/65 = 72/C
To find A, we take the following ratios.
A/13 = 45/65
On solving, we get
A = 45 * 13 / 65 = 585/65 = 9
To find B, we take the following ratios.
18/B = 45/65
B = 18 * 65 / 45 = 1170/45 = 26
To find C, we take the following ratios.
72/C = 45/65
C = 72 * 65 / 45 = 104
Therefore using the given ratios we can complete the table using the values of A, B and C,
Black 9 18 45 72
White 13 26 65 104
To learn more about ratios, follow the link.
https://brainly.com/question/2328454
#SPJ1
If it is windy, then we will go sailing today.
A. If we go sailing today, then it is windy.
B. If it is windy, then we will not go sailing today.
C. If we do not go sailing today, then it is not windy.
D.If it is not windy, then we will not go sailing today.
The probability that the San Jose Sharks will win any given game is 0.3694 based on a 13-year win history of 382 wins out of 1,034 games played (as of a certain date). An upcoming monthly schedule contains 12 games.
a. What is the expected number of wins for the upcoming month? Let X = number of games won in November 2005.
b. Suppose X = N(3, 2). What value of x is two standard deviations to the right of the mean? (Enter an exact number as an integer, fraction, or decimal.)
c. Suppose X ∼ N(18, 3). Between what x values does 68.27% of the data lie? The range of x values is centered at the mean of the distribution (i.e., 18).
A. Between x = -3 and x = 3
B. Between x = 12 and x = 24
C. Between x = -17 and x = 18
D. Between x = 9 and x = 27
E. Between x = 15 and x = 21
.
The probability that the San Jose Sharks will win any given game is 0.3694 based on a 13-year win history of 382 wins out of 1,034 games played (as of a certain date).
An upcoming monthly schedule contains 12 games.a. Expected number of wins for the upcoming month:
Let X = number of games won in November 2005.The number of wins in a given game follows a Bernoulli distribution with probability of success p = 0.3694 and q = 1 - p = 0.6306.The expected number of wins in n Bernoulli trials is np.E(X) = npExpected number of wins for the upcoming month E(X) = 12 × 0.3694 = 4.4328
Therefore, the expected number of wins for the upcoming month is 4.4328.b. Suppose X = N(3, 2).What value of x is two standard deviations to the right of the mean? (Enter an exact number as an integer, fraction, or decimal.)
We have, μ = 3 and σ = 2Standard deviation is defined as the square root of the variance.The variance of the normal distribution is σ² = 2² = 4.Two standard deviations to the right of the mean means a z-score of 2.The z-score is calculated using the formula:
z = (x - μ) / σ2
= (x - 3) / 2x - 3
= 4x
= 7
Hence, the value of x is 7.c. Suppose X ∼ N(18, 3).
Between what x values does 68.27% of the data lie?We have, μ = 18 and σ = 3.68.27% of the data is within one standard deviation of the mean, which means the z-score is between -1 and 1.Using the formula: z = (x - μ) / σFor the lower limit, z = -1 -1 = (x - 18) / 3x = -3 + 18 = 15For the upper limit, z = 1 1 = (x - 18) / 3x = 3 + 18 = 21Hence, the data lies between x = 15 and x = 21.Therefore, the Answer is Option E: Between x = 15 and x = 21.
In summary, the expected number of wins for the upcoming month for San Jose Sharks is 4.4328. For X = N(3,2), the value of x is 7. For X ∼ N(18, 3), the data lies between x = 15 and x = 21.
Learn more about Bernoulli distribution here:
brainly.com/question/32129510
#SPJ11
Question 5 of 100. Marty (62), single, has 2022 taxable income of $510,000. What is Marty's marginal tax rate?
35%
37%
38.5%
39.6%
Marty's taxable income of $510,000 falls within the last tax bracket, his marginal tax rate would be 37%.
To determine Marty's marginal tax rate, we need to refer to the tax brackets for the given year. However, as my knowledge is based on information up until September 2021, I can provide you with the tax brackets for that year. Please note that tax laws may change, so it is always best to consult the current tax regulations or a tax professional for accurate information.
For the 2021 tax year, the marginal tax rates for individuals are as follows:
10% on taxable income up to $9,950
12% on taxable income between $9,951 and $40,525
22% on taxable income between $40,526 and $86,375
24% on taxable income between $86,376 and $164,925
32% on taxable income between $164,926 and $209,425
35% on taxable income between $209,426 and $523,600
37% on taxable income over $523,600
Since Marty's taxable income of $510,000 falls within the last tax bracket, his marginal tax rate would be 37%. However, please note that tax rates can vary based on changes in tax laws and regulations, so it's essential to consult the current tax laws or a tax professional for the most accurate information.
To learn more about marginal tax rate
https://brainly.com/question/29998903
#SPJ11
UNIT 5: CEREAL BOX PROJECT / PORTFOLIO
HELP
Using the formula for volume and surface area of solids, we have the following:
1. 180 in.³; 258 in.²; $12.9
2. 144 in.³; 184 in.²; $9.2
3. 75 in.³; 228 in.²; $11.4
What is Volume and Surface Area?Volume of a rectangular prism = l × w × h
Surface area of a rectangular prism = 2(wl+hl+hw)
Volume of a square pyramid = 1/3 × a² × h
Surface area of a square pyramid = a² + 2al
Volume of a cylinder = πr²h
Surface area of a cylinder = 2πr( h + r)
1. Volume of the rectangular prism cereal box = 7.5 × 2 × 12 = 180 in.³
Surface area of the rectangular prism cereal box = 2(wl+hl+hw) = 2(2 × 7.5 + 12 × 7.5 + 12 × 2) = 258 in.²
Cost of material for the the rectangular prism cereal box = 258 × 0.05 = $12.9.
2. Volume of the square pyramid cereal box = 1/3 × a² × h = 1/3 × 6² × 12 = 144 in.³
Surface area of the square pyramid cereal box = a² + 2al = 6² + 2(6)(12.3) ≈ 184 in.²
Cost of material for the the square pyramid cereal box = 184 × 0.05 = $9.2.
3. Volume of the cylinder cereal box = πr²h = π(2.5²)(12) = 75 in.³
Surface area of the cylinder cereal box = 2π(2.5)(12 + 2.5) ≈ 228 in.²
Cost of material for the the cylinder cereal box = 228 × 0.05 = $11.4.
Learn more about the surface area and volume of solids on:
https://brainly.com/question/16599646
In a large consumer survey, 610 randomly selected customers are interviewed in a large grocery store. They are asked, among other things, how much they have shopped for. On average, they have shopped for 412 SEK with a standard deviation of 181 SEK . Estimate with a 95% confidence interval the average purchase amount in the entire population.
Answer only with the statistical margin of error.
On the arranged survey, the statistical margin of error is supposed to be 14.73 SEK.
In a large consumer survey, 610 randomly selected customers are interviewed in a large grocery store. They are asked, among other things, how much they have shopped for. On average, they have shopped for 412 SEK with a standard deviation of 181 SEK .
Estimate with a 95% confidence interval the average purchase amount in the entire population. The formula for finding the margin of error is given as;
Margin of Error = z * (σ/√n)
Where:
σ is the standard deviation
n is the sample size
z is the Z-score for the desired confidence level, which is 95%.
Z-score corresponding to 95% confidence level is 1.96
Margin of Error = 1.96 * (181/√610) = 14.73
The statistical margin of error is 14.73 SEK.
Learn more about Statistical Margin of Error here: brainly.com/question/30760735
#SPJ11
What is the product of the polynomials below?
(5x2 - x-3)(2x+6)
A. 10x3 + 28x2 + 12x +18
B. 10x2+28x2 + 12x +3
O C. 10x2 +28x2 - 12x-18
D. 10x2 +28x2 - 12x-3
I NEED HELP PLZ (no links!)
Name the planet (including Pluto) in order from smallest to largest
Answer:
Mecury
Mars
Venus
Earth
Neptune
Uranus
Saturn
Jupiter
At 6 a.m., a meteorologist records the temperature as −3.2°F . Seven hours later, the meteorologist records the temperature as 11.7°F.
Which expression represents the total temperature change, in degrees Fahrenheit, over the seven hours?
11.7+(−3.2)
−3.2+11.7
|11.7+(−3.2)|
|11.7−(−3.2)|
The correct expression in this case is |11.7−(−3.2)|.
Negative numbersGiven that at 6 am, a meteorologist records the temperature as −3.2°F, and seven hours later, the meteorologist records the temperature as 11.7°F, to determine which expression represents the total temperature change, in degrees Fahrenheit, over the seven hours, the following reasoning must be carried out:
A negative number is a value that is below 0, that is, the inverse of said negative number is required in positive values to equate to 0.So, -3.2 requires 3.2 to get to 0. If we add 11.7 to that, the total required values will be 14.9 (3.2 + 11.7).In turn, in mathematics, subtracting a negative number from a positive number implies adding that number (1 - (-1) = 2).Therefore, the correct expression in this case is |11.7−(−3.2)|.
Learn more about negative numbers in https://brainly.com/question/258076
Someone plz help me :(
Answer: 6 feet
Step-by-step explanation: 12 x6 =72 or 72/12 =6
Answer the question below. Type your response in the space provided.
What is the surface area of the pyramid formed from the net shown here? The triangles are
equilateral, and each triangle has a height of 5.2 centimeters. Write your answer rounded to one
decimal point and with the correct unit.
6 cm
Done
Clear
6 cm
6 cm
Answer:
\(62.4$ cm^2\)
Step-by-step explanation:
The net of the triangular pyramid is attached below.
Since each of the faces is an equilateral triangle of side 6cm, the base of the pyramid will also be an equilateral triangle of side lengths 6cm .
In the equilateral triangles
Base = 6cmHeight =5.2 cmTherefore, the surface area of the pyramid formed from the net shown
= 4 X Area of One Triangular Face
\(=4 (\frac12 \times bh)\\=4 (\frac12 \times 6\times 5.2)\\=4(15.6)\\=62.4$ cm^2\)
Ver en español
Gary is exercising on the stationary bike in his garage. He bikes 3. 5 miles in
1
4
of an hour. How many miles could Gary bike in
3
4
of an hour at the same speed?
Gary podría recorrer 8,5 millas en 34 minutos a la misma velocidad.
Ver en españolGary está haciendo ejercicio en la bicicleta estática de su garaje. Pedalea 3,5 millas en 14 de una hora. ¿Cuántas millas podría pedalear Gary en 34 de una hora a la misma velocidad?
Gary bicicletas a 3.5 / 1 = 0.25 millas por minuto.La velocidad de
Gary en millas por minuto = (Distancia) / (Tiempo)Entonces,
la velocidad de Gary = 3.5 millas / 14 minutos= 0.25 millas por minutoPor lo tanto,
podemos encontrar la distancia que recorrerá Gary en 34 minutos usando la velocidad que se calculó anteriormente. La distancia que recorrerá Gary en 34 minutos es:
Distancia = Velocidad * Tiempo
Distancia = 0.25 millas / minuto * 34 minutos=
8.5 millasGary podría recorrer 8,5 millas en 34 minutos a la misma velocidad. Entonces, la respuesta es 8,5. Por lo tanto, podemos encontrar la distancia que recorrerá Gary en 34 minutos usando la velocidad que se calculó anteriormente.
To learn more about Velocidad :
https://brainly.com/question/28923504
#SPJ11
which is a mathematical property shown here
8(x+7)
1 commutative property
2 distributive property
3 identity of addition